Difference between revisions of "2011 AMC 12A Problems/Problem 2"

m (Solution)
(Solution)
Line 17: Line 17:
  
 
== Solution ==
 
== Solution ==
By careful inspection, the answer is <math>\textbf{(E)}</math>.
+
By careful inspection and common sense, the answer is <math>\textbf{(E)}</math>.
  
 
== See also ==
 
== See also ==

Revision as of 15:38, 24 June 2016

Problem

There are $5$ coins placed flat on a table according to the figure. What is the order of the coins from top to bottom? [asy] size(100); defaultpen(linewidth(.8pt)+fontsize(8pt)); draw(arc((0,1), 1.2, 25, 214)); draw(arc((.951,.309), 1.2, 0, 360)); draw(arc((.588,-.809), 1.2, 132, 370)); draw(arc((-.588,-.809), 1.2, 75, 300)); draw(arc((-.951,.309), 1.2, 96, 228)); label("$A$",(0,1),NW); label("$B$",(-1.1,.309),NW); label("$C$",(.951,.309),E); label("$D$",(-.588,-.809),W); label("$E$",(.588,-.809),S);[/asy] $\textbf{(A)}\ (C, A, E, D, B) \qquad \textbf{(B)}\ (C, A, D, E, B) \qquad \textbf{(C)}\ (C, D, E, A, B) \qquad \textbf{(D)}\ (C, E, A, D, B) \qquad \\ \textbf{(E)}\ (C, E, D, A, B)$

Solution

By careful inspection and common sense, the answer is $\textbf{(E)}$.

See also

2011 AMC 12A (ProblemsAnswer KeyResources)
Preceded by
Problem 1
Followed by
Problem 3
1 2 3 4 5 6 7 8 9 10 11 12 13 14 15 16 17 18 19 20 21 22 23 24 25
All AMC 12 Problems and Solutions

The problems on this page are copyrighted by the Mathematical Association of America's American Mathematics Competitions. AMC logo.png